수학 문풀(20)
-
RMM11 P4
문제 링크 Community Something appears to not have loaded correctly. artofproblemsolving.com 문제. n=p1e1p2e2⋯pkekn = p_{1}^{e_{1}}p_{2}^{e_{2}} \cdots p_{k}^{e_{k}}n=p1e1p2e2⋯pkek에 대해 λ(n)\lambda(n)λ(n)은 (−1)e1+e2+⋯ek(-1)^{e_1 + e_2 + \cdots e_k}(−1)e1+e2+⋯ek로 정의된다. (Liouville's function) 다음을 증명하여라: (a) λ(n)=λ(n+1)=1\lambda(n) = \lambda(n+1) = 1λ(n)=λ(n+1)=1인 nnn이 무한히 많다. (b) λ(n)=λ(n+1)=−1\lambda(n) = \lambda(n+1) = -1λ(n)=λ(n+1)=−1인 nnn이 무한히 많다. 아이디어가 상당히 기초적이면서도 재밌다. 풀이. ...더보기 (a) 편의를 위해, ..
2019.04.23 -
KAIST POW 2019-03 Simple Spectrum
문제 링크 2019-03 Simple spectrum - KAIST Math Problem of the Week Suppose that T T T is an N×N N \times N N×N matrix \[ T = \begin{pmatrix} a_1 & b_1 & 0 & \cdots & 0 \\ b_1 & a_2 & b_2 & \ddots & \vdots \\ 0 & b_2 & a_3 & \ddots & 0 \\ \vdots & \ddots & \ddots & \ddots & … mathsci.kaist.ac.kr N×NN \times NN×N 실수 행렬 TTT가 다음과 같고, 모든 bi>0b_i > 0bi>0일 때 TTT는 서로 다른 NNN개의 eigenvalue를 가짐을 보여라. T = \be..
2019.03.29 -
KAIST POW 2019-02 Simplification of an expression with factorials
문제 링크 2019-02 Simplification of an expression with factorials - KAIST Math Problem of the Week For any positive integers m and n, show that Cn,m=(mn)!(m!)nn! C_{n,m} = \frac{(mn)!}{(m!)^n n!} Cn,m=(m!)nn!(mn)! is an integer. Recommend on Facebook Tweet about it Print for later Tell a friend Related mathsci.kaist.ac.kr POW라는 걸 믿을 수 없을 정도로 쉬운 문제가 나와버렸다. 스포방지 안할래... 문제. 모든 자연수 n,mn,mn,m에 대해, 다음이 정수임을 보여라. C_{n,m} = \frac{(nm)!..
2019.03.29 -
Balkan MO 2015
문제 링크 재활을 위해 IMO보다 부담 없이 건드려볼 수 있는 셋을 선택한다고 했는데... 아직 나한텐 너무 어렵다.문제 셋은 굉장히 좋다. 고인물 테크닉을 요구하지도 않고.나는 4번만 풀이를 봤고, 123은 혼자서 풀었다.스포방지선 1번 (대수) 그사건... x=ab2,y=bc2,z=ca2x = ab^2, y = bc^2, z = ca^2x=ab2,y=bc2,z=ca2으로 치환하면 n=1n = 1n=1 Schur가 된다.어차피 부등식은 이제 나오지 않으니 던져버리자! 2번 (기하) 지오지브라의 힘을 빌렸다. 점을 몇 개 더 잡지만 저 그림을 지워버린 관계로 말로 때우자.잘 그린 그림으로부터, MI(∥AB)MI (\parallel AB)MI(∥AB), ACACAC, EFEFEF가 한 점에서 만난다는 것을 알 수 있다. Lemma) U=EF∩ACU = EF \cap ACU=EF∩AC, \(V = ..
2019.02.09 -
Putnam 2017 풀이 - 풀리는 것만
AoPS 링크 A1. 다음을 만족하는 최소의 집합 SSS에 속하지 않는 원소로는 어떤 것이 있겠는가?- 2∈S2 \in S2∈S.- n2∈S ⟹ n∈Sn^2 \in S \implies n \in Sn2∈S⟹n∈S.- n∈S ⟹ (n+5)2∈Sn \in S \implies (n+5)^2 \in Sn∈S⟹(n+5)2∈S. 답은 1,5k1, 5k1,5k.나는 그냥 bound 줄여나가면서 풀었는데, 훨씬 깔끔한 풀이가 있어서 가져왔다. n∈Sn \in Sn∈S에 대해서 n+5∈Sn + 5 \in Sn+5∈S가 자명하다. 또 2가 원소면 49가 원소고, 542=291654^2 = 2916542=2916이 SSS의 원소다. 이는 5k+15k+15k+1꼴이고, 따라서 291629162916 이상의 5k+15k+15k+1 꼴은 모두 SSS에 포함된다.임의의 5∤x5 \not{|} x5∣x에 대해 \(x^{16} \equiv 1 \ (\mod..
2018.11.30 -
2018 대수경 2분야 5번 풀이
문제. 1⋯n1 \cdots n1⋯n의 순열 a1,a2,…ana_{1}, a_{2}, \ldots a_{n}a1,a2,…an과 b1,b2,…bnb_{1}, b_{2}, \ldots b_{n}b1,b2,…bn에 대해, 다음 행렬 AAA의 행렬식으로 가능한 값을 모두 구하여라. A=(Aij)=((1+aibj)n−1) A = (A_{ij}) = ((1+a_{i}b_{j})^{n-1}) A=(Aij)=((1+aibj)n−1) n=2n = 2n=2일 때는 행렬을 다음의 꼴로 나타낼 수 있다. A=(1a11a2)⋅(11b1b2) A = \begin{pmatrix} 1 & a_{1} \\ 1 & a_{2} \end{pmatrix} \cdot \begin{pmatrix} 1 & 1 \\ b_{1} & b_{2} \end{pmatrix} A=(11a1a2)⋅(1b11b2) 따라서 적당한 k∈{0,1}k \in \{0,1\}k∈{0,1}에 대해 \det (A) = (-1)^{k} \det \left[ \begin..
2018.11.24